GET READY FOR THE AMC 10 WITH AoPS
Learn with outstanding instructors and top-scoring students from around the world in our AMC 10 Problem Series online course.
CHECK SCHEDULE

Difference between revisions of "2022 AMC 10A Problems"

 
(141 intermediate revisions by 26 users not shown)
Line 1: Line 1:
 
{{AMC10 Problems|year=2022|ab=A}}
 
{{AMC10 Problems|year=2022|ab=A}}
 
These are the 2020 problems. Please edit the correct problems in.
 
  
 
==Problem 1==
 
==Problem 1==
  
What value of <math>x</math> satisfies
+
What is the value of <cmath>3+\frac{1}{3+\frac{1}{3+\frac13}}?</cmath>
 
+
<math>\textbf{(A)}\ \frac{31}{10}\qquad\textbf{(B)}\ \frac{49}{15}\qquad\textbf{(C)}\ \frac{33}{10}\qquad\textbf{(D)}\ \frac{109}{33}\qquad\textbf{(E)}\ \frac{15}{4}</math>
<cmath>x- \frac{3}{4} = \frac{5}{12} - \frac{1}{3}?</cmath>
 
<math>\textbf{(A)}\ -\frac{2}{3}\qquad\textbf{(B)}\ \frac{7}{36}\qquad\textbf{(C)}\ \frac{7}{12}\qquad\textbf{(D)}\ \frac{2}{3}\qquad\textbf{(E)}\ \frac{5}{6}</math>
 
  
[[2020 AMC 10A Problems/Problem 1|Solution]]
+
[[2022 AMC 10A Problems/Problem 1|Solution]]
  
 
==Problem 2==
 
==Problem 2==
  
The numbers <math>3, 5, 7, a,</math> and <math>b</math> have an average (arithmetic mean) of <math>15</math>. What is the average of <math>a</math> and <math>b</math>?
+
Mike cycled <math>15</math> laps in <math>57</math> minutes. Assume he cycled at a constant speed throughout. Approximately how many laps did he complete in the first <math>27</math> minutes?
  
<math>\textbf{(A) } 0 \qquad\textbf{(B) } 15 \qquad\textbf{(C) } 30 \qquad\textbf{(D) } 45 \qquad\textbf{(E) } 60</math>
+
<math>\textbf{(A) } 5 \qquad\textbf{(B) } 7 \qquad\textbf{(C) } 9 \qquad\textbf{(D) } 11 \qquad\textbf{(E) } 13</math>
  
[[2020 AMC 10A Problems/Problem 2|Solution]]
+
[[2022 AMC 10A Problems/Problem 2|Solution]]
  
 
==Problem 3==
 
==Problem 3==
  
Assuming <math>a\neq3</math>, <math>b\neq4</math>, and <math>c\neq5</math>, what is the value in simplest form of the following expression?
+
The sum of three numbers is <math>96.</math> The first number is <math>6</math> times the third number, and the third number is <math>40</math> less than the second number. What is the absolute value of the difference between the first and second numbers?
<cmath>\frac{a-3}{5-c} \cdot \frac{b-4}{3-a} \cdot \frac{c-5}{4-b}</cmath>
 
  
<math>\textbf{(A) } -1 \qquad \textbf{(B) } 1 \qquad \textbf{(C) } \frac{abc}{60} \qquad \textbf{(D) } \frac{1}{abc} - \frac{1}{60} \qquad \textbf{(E) } \frac{1}{60} - \frac{1}{abc}</math>
+
<math>\textbf{(A) } 1 \qquad \textbf{(B) } 2 \qquad \textbf{(C) } 3 \qquad \textbf{(D) } 4 \qquad \textbf{(E) } 5</math>
  
[[2020 AMC 10A Problems/Problem 3|Solution]]
+
[[2022 AMC 10A Problems/Problem 3|Solution]]
  
 
==Problem 4==
 
==Problem 4==
  
A driver travels for <math>2</math> hours at <math>60</math> miles per hour, during which her car gets <math>30</math> miles per gallon of gasoline. She is paid <math>\$0.50</math> per mile, and her only expense is gasoline at <math>\$2.00</math> per gallon. What is her net rate of pay, in dollars per hour, after this expense?
+
In some countries, automobile fuel efficiency is measured in liters per <math>100</math> kilometers while other countries use miles per gallon. Suppose that 1 kilometer equals <math>m</math> miles, and <math>1</math> gallon equals <math>l</math> liters. Which of the following gives the fuel efficiency in liters per <math>100</math> kilometers for a car that gets <math>x</math> miles per gallon?
  
<math>\textbf{(A) }20 \qquad\textbf{(B) }22 \qquad\textbf{(C) }24 \qquad\textbf{(D) } 25\qquad\textbf{(E) } 26</math>
+
<math>\textbf{(A) } \frac{x}{100lm} \qquad \textbf{(B) } \frac{xlm}{100} \qquad \textbf{(C) } \frac{lm}{100x} \qquad \textbf{(D) } \frac{100}{xlm} \qquad \textbf{(E) } \frac{100lm}{x}</math>
  
[[2020 AMC 10A Problems/Problem 4|Solution]]
+
[[2022 AMC 10A Problems/Problem 4|Solution]]
  
 
==Problem 5==
 
==Problem 5==
  
What is the sum of all real numbers <math>x</math> for which <math>|x^2-12x+34|=2?</math>
+
Square <math>ABCD</math> has side length <math>1</math>. Points <math>P</math>, <math>Q</math>, <math>R</math>, and <math>S</math> each lie on a side of <math>ABCD</math> such that <math>APQCRS</math> is an equilateral convex hexagon with side length <math>s</math>. What is <math>s</math>?
  
<math>\textbf{(A) } 12 \qquad \textbf{(B) } 15 \qquad \textbf{(C) } 18 \qquad \textbf{(D) } 21 \qquad \textbf{(E) } 25</math>
+
<math>\textbf{(A) } \frac{\sqrt{2}}{3} \qquad \textbf{(B) } \frac{1}{2} \qquad \textbf{(C) } 2 - \sqrt{2} \qquad \textbf{(D) } 1 - \frac{\sqrt{2}}{4} \qquad \textbf{(E) } \frac{2}{3}</math>
  
[[2020 AMC 10A Problems/Problem 5|Solution]]
+
[[2022 AMC 10A Problems/Problem 5|Solution]]
  
 
==Problem 6==
 
==Problem 6==
  
How many <math>4</math>-digit positive integers (that is, integers between <math>1000</math> and <math>9999</math>, inclusive) having only even digits are divisible by <math>5?</math>
+
Which expression is equal to <cmath>\left|a-2-\sqrt{(a-1)^2}\right|</cmath> for <math>a<0?</math>
  
<math>\textbf{(A) } 80 \qquad \textbf{(B) } 100 \qquad \textbf{(C) } 125 \qquad \textbf{(D) } 200 \qquad \textbf{(E) } 500</math>
+
<math>\textbf{(A) } 3-2a \qquad \textbf{(B) } 1-a \qquad \textbf{(C) } 1 \qquad \textbf{(D) } a+1 \qquad \textbf{(E) } 3</math>
  
[[2020 AMC 10A Problems/Problem 6|Solution]]
+
[[2022 AMC 10A Problems/Problem 6|Solution]]
  
 
==Problem 7==
 
==Problem 7==
  
The <math>25</math> integers from <math>-10</math> to <math>14,</math> inclusive, can be arranged to form a <math>5</math>-by-<math>5</math> square in which the sum of the numbers in each row, the sum of the numbers in each column, and the sum of the numbers along each of the main diagonals are all the same. What is the value of this common sum?
+
The least common multiple of a positive integer <math>n</math> and <math>18</math> is <math>180</math>, and the greatest common divisor of <math>n</math> and <math>45</math> is <math>15</math>. What is the sum of the digits of <math>n</math>?
  
<math>\textbf{(A) }2 \qquad\textbf{(B) } 5\qquad\textbf{(C) } 10\qquad\textbf{(D) } 25\qquad\textbf{(E) } 50</math>
+
<math>\textbf{(A) } 3 \qquad \textbf{(B) } 6 \qquad \textbf{(C) } 8 \qquad \textbf{(D) } 9 \qquad \textbf{(E) } 12</math>
  
[[2020 AMC 10A Problems/Problem 7|Solution]]
+
[[2022 AMC 10A Problems/Problem 7|Solution]]
  
 
==Problem 8==
 
==Problem 8==
  
What is the value of
+
A data set consists of <math>6</math> (not distinct) positive integers: <math>1</math>, <math>7</math>, <math>5</math>, <math>2</math>, <math>5</math>, and <math>X</math>. The average (arithmetic mean) of the <math>6</math> numbers equals a value in the data set. What is the sum of all possible values of <math>X</math>?
  
<cmath>1+2+3-4+5+6+7-8+\cdots+197+198+199-200?</cmath>
+
<math>\textbf{(A) } 10 \qquad \textbf{(B) } 26 \qquad \textbf{(C) } 32 \qquad \textbf{(D) } 36 \qquad \textbf{(E) } 40</math>
  
<math>\textbf{(A) } 9,800 \qquad \textbf{(B) } 9,900 \qquad \textbf{(C) } 10,000 \qquad \textbf{(D) } 10,100 \qquad \textbf{(E) } 10,200</math>
+
[[2022 AMC 10A Problems/Problem 8|Solution]]
  
[[2020 AMC 10A Problems/Problem 8|Solution]]
+
==Problem 9==
  
==Problem 9==
+
A rectangle is partitioned into <math>5</math> regions as shown. Each region is to be painted a solid color - red, orange, yellow, blue, or green - so that regions that touch are painted different colors, and colors can be used more than once. How many different colorings are possible?
  
A single bench section at a school event can hold either <math>7</math> adults or <math>11</math> children. When <math>N</math> bench sections are connected end to end, an equal number of adults and children seated together will occupy all the bench space. What is the least possible positive integer value of <math>N?</math>
+
<asy> size(5.5cm); draw((0,0)--(0,2)--(2,2)--(2,0)--cycle); draw((2,0)--(8,0)--(8,2)--(2,2)--cycle); draw((8,0)--(12,0)--(12,2)--(8,2)--cycle); draw((0,2)--(6,2)--(6,4)--(0,4)--cycle); draw((6,2)--(12,2)--(12,4)--(6,4)--cycle); </asy>
  
<math>\textbf{(A) } 9 \qquad \textbf{(B) } 18 \qquad \textbf{(C) } 27 \qquad \textbf{(D) } 36 \qquad \textbf{(E) } 77</math>
+
<math>\textbf{(A) }120\qquad\textbf{(B) }270\qquad\textbf{(C) }360\qquad\textbf{(D) }540\qquad\textbf{(E) }720</math>
  
[[2020 AMC 10A Problems/Problem 9|Solution]]
+
[[2022 AMC 10A Problems/Problem 9|Solution]]
  
 
==Problem 10==
 
==Problem 10==
  
Seven cubes, whose volumes are <math>1</math>, <math>8</math>, <math>27</math>, <math>64</math>, <math>125</math>, <math>216</math>, and <math>343</math> cubic units, are stacked vertically to form a tower in which the volumes of the cubes decrease from bottom to top. Except for the bottom cube, the bottom face of each cube lies completely on top of the cube below it. What is the total surface area of the tower (including the bottom) in square units?
+
Daniel finds a rectangular index card and measures its diagonal to be <math>8</math> centimeters.
 +
Daniel then cuts out equal squares of side <math>1</math> cm at two opposite corners of the index card and measures the distance between the two closest vertices of these squares to be <math>4\sqrt{2}</math> centimeters, as shown below. What is the area of the original index card?
 +
<asy>
 +
// Diagram by MRENTHUSIASM, edited by Djmathman
 +
size(200);
 +
defaultpen(linewidth(0.6));
 +
draw((489.5,-213) -- (225.5,-213) -- (225.5,-185) -- (199.5,-185) -- (198.5,-62) -- (457.5,-62) -- (457.5,-93) -- (489.5,-93) -- cycle);
 +
draw((206.29,-70.89) -- (480.21,-207.11), linetype ("6 6"),Arrows(size=4,arrowhead=HookHead));
 +
draw((237.85,-182.24) -- (448.65,-95.76),linetype ("6 6"),Arrows(size=4,arrowhead=HookHead));
 +
label("$1$",(450,-80));
 +
label("$1$",(475,-106));
 +
label("$8$",(300,-103));
 +
label("$4\sqrt 2$",(300,-173));
 +
</asy>
 +
<math>\textbf{(A) } 14 \qquad \textbf{(B) } 10\sqrt{2} \qquad \textbf{(C) } 16 \qquad \textbf{(D) } 12\sqrt{2} \qquad \textbf{(E) } 18</math>
  
<math>\textbf{(A)}\ 644\qquad\textbf{(B)}\ 658\qquad\textbf{(C)}\ 664\qquad\textbf{(D)}\ 720\qquad\textbf{(E)}\ 749</math>
+
[[2022 AMC 10A Problems/Problem 10|Solution]]
  
[[2020 AMC 10A Problems/Problem 10|Solution]]
+
==Problem 11==
  
==Problem 11==
+
Ted mistakenly wrote <math>2^m\cdot\sqrt{\frac{1}{4096}}</math> as <math>2\cdot\sqrt[m]{\frac{1}{4096}}.</math> What is the sum of all real numbers <math>m</math> for which these two expressions have the same value?
  
What is the median of the following list of <math>4040</math> numbers<math>?</math>
+
<math>\textbf{(A) } 5 \qquad \textbf{(B) } 6 \qquad \textbf{(C) } 7 \qquad \textbf{(D) } 8 \qquad \textbf{(E) } 9</math>
<cmath>1, 2, 3, \ldots, 2020, 1^2, 2^2, 3^2, \ldots, 2020^2</cmath>
 
<math> \textbf{(A)}\ 1974.5\qquad\textbf{(B)}\ 1975.5\qquad\textbf{(C)}\ 1976.5\qquad\textbf{(D)}\ 1977.5\qquad\textbf{(E)}\ 1978.5 </math>
 
  
[[2020 AMC 10A Problems/Problem 11|Solution]]
+
[[2022 AMC 10A Problems/Problem 11|Solution]]
  
 
==Problem 12==
 
==Problem 12==
  
Triangle <math>AMC</math> is isosceles with <math>AM = AC</math>. Medians <math>\overline{MV}</math> and <math>\overline{CU}</math> are perpendicular to each other, and <math>MV=CU=12</math>. What is the area of <math>\triangle AMC?</math>
+
On Halloween <math>31</math> children walked into the principal's office asking for candy. They
 +
can be classified into three types: Some always lie; some always tell the truth; and
 +
some alternately lie and tell the truth. The alternaters arbitrarily choose their first
 +
response, either a lie or the truth, but each subsequent statement has the opposite
 +
truth value from its predecessor. The principal asked everyone the same three
 +
questions in this order.
  
<asy>
+
"Are you a truth-teller?" The principal gave a piece of candy to each of the <math>22</math>
draw((-4,0)--(4,0)--(0,12)--cycle);
+
children who answered yes.
draw((-2,6)--(4,0));
+
 
draw((2,6)--(-4,0));
+
"Are you an alternater?" The principal gave a piece of candy to each of the <math>15</math>
label("M", (-4,0), W);
+
children who answered yes.
label("C", (4,0), E);
+
 
label("A", (0, 12), N);
+
"Are you a liar?" The principal gave a piece of candy to each of the <math>9</math> children who
label("V", (2, 6), NE);
+
answered yes.
label("U", (-2, 6), NW);
+
 
label("P", (0, 3.6), S);
+
How many pieces of candy in all did the principal give to the children who always
</asy>
+
tell the truth?
  
<math>\textbf{(A) } 48 \qquad \textbf{(B) } 72 \qquad \textbf{(C) } 96 \qquad \textbf{(D) } 144 \qquad \textbf{(E) } 192</math>
+
<math>\textbf{(A) } 7 \qquad \textbf{(B) } 12 \qquad \textbf{(C) } 21 \qquad \textbf{(D) } 27 \qquad \textbf{(E) } 31</math>
  
[[2020 AMC 10A Problems/Problem 12|Solution]]
+
[[2022 AMC 10A Problems/Problem 12|Solution]]
  
 
==Problem 13==
 
==Problem 13==
  
A frog sitting at the point <math>(1, 2)</math> begins a sequence of jumps, where each jump is parallel to one of the coordinate axes and has length <math>1</math>, and the direction of each jump (up, down, right, or left) is chosen independently at random. The sequence ends when the frog reaches a side of the square with vertices <math>(0,0), (0,4), (4,4),</math> and <math>(4,0)</math>. What is the probability that the sequence of jumps ends on a vertical side of the square?
+
Let <math>\triangle ABC</math> be a scalene triangle. Point <math>P</math> lies on <math>\overline{BC}</math> so that <math>\overline{AP}</math> bisects <math>\angle BAC.</math> The line through <math>B</math> perpendicular to <math>\overline{AP}</math> intersects the line through <math>A</math> parallel to <math>\overline{BC}</math> at point <math>D.</math> Suppose <math>BP=2</math> and <math>PC=3.</math> What is <math>AD?</math>
  
<math>\textbf{(A)}\ \frac12\qquad\textbf{(B)}\ \frac 58\qquad\textbf{(C)}\ \frac 23\qquad\textbf{(D)}\ \frac34\qquad\textbf{(E)}\ \frac 78</math>
+
<math>\textbf{(A) } 8 \qquad \textbf{(B) } 9 \qquad \textbf{(C) } 10 \qquad \textbf{(D) } 11 \qquad \textbf{(E) } 12</math>
  
[[2020 AMC 10A Problems/Problem 13|Solution]]
+
[[2022 AMC 10A Problems/Problem 13|Solution]]
  
 
==Problem 14==
 
==Problem 14==
  
Real numbers <math>x</math> and <math>y</math> satisfy <math>x + y = 4</math> and <math>x \cdot y = -2</math>. What is the value of<cmath>x + \frac{x^3}{y^2} + \frac{y^3}{x^2} + y?</cmath><math>\textbf{(A)}\ 360\qquad\textbf{(B)}\ 400\qquad\textbf{(C)}\ 420\qquad\textbf{(D)}\ 440\qquad\textbf{(E)}\ 480</math>
+
How many ways are there to split the integers <math>1</math> through <math>14</math> into <math>7</math> pairs such that in each pair, the greater number is at least <math>2</math> times the lesser number?
  
[[2020 AMC 10A Problems/Problem 14|Solution]]
+
<math>\textbf{(A) } 108 \qquad \textbf{(B) } 120 \qquad \textbf{(C) } 126 \qquad \textbf{(D) } 132 \qquad \textbf{(E) } 144</math>
 +
 
 +
[[2022 AMC 10A Problems/Problem 14|Solution]]
  
 
==Problem 15==
 
==Problem 15==
  
A positive integer divisor of <math>12!</math> is chosen at random. The probability that the divisor chosen is a perfect square can be expressed as <math>\frac{m}{n}</math>, where <math>m</math> and <math>n</math> are relatively prime positive integers. What is <math>m+n</math>?
+
Quadrilateral <math>ABCD</math> with side lengths <math>AB=7, BC=24, CD=20, DA=15</math> is inscribed in a circle. The area interior to the circle but exterior to the quadrilateral can be written in the form <math>\frac{a\pi-b}{c},</math> where <math>a,b,</math> and <math>c</math> are positive integers such that <math>a</math> and <math>c</math> have no common prime factor. What is <math>a+b+c?</math>
  
<math>\textbf{(A)}\ 3\qquad\textbf{(B)}\ 5\qquad\textbf{(C)}\ 12\qquad\textbf{(D)}\ 18\qquad\textbf{(E)}\ 23</math>
+
<math>\textbf{(A) } 260 \qquad \textbf{(B) } 855 \qquad \textbf{(C) } 1235 \qquad \textbf{(D) } 1565 \qquad \textbf{(E) } 1997</math>
  
[[2020 AMC 10A Problems/Problem 15|Solution]]
+
[[2022 AMC 10A Problems/Problem 15|Solution]]
  
 
==Problem 16==
 
==Problem 16==
  
A point is chosen at random within the square in the coordinate plane whose vertices are <math>(0, 0), (2020, 0), (2020, 2020),</math> and <math>(0, 2020)</math>. The probability that the point is within <math>d</math> units of a lattice point is <math>\tfrac{1}{2}</math>. (A point <math>(x, y)</math> is a lattice point if <math>x</math> and <math>y</math> are both integers.) What is <math>d</math> to the nearest tenth?
+
The roots of the polynomial <math>10x^3 - 39x^2 + 29x - 6</math> are the height, length, and width of a rectangular box (right rectangular prism). A new rectangular box is formed by lengthening each edge of the original box by <math>2</math> units. What is the volume of the new box?
  
<math>\textbf{(A) } 0.3 \qquad \textbf{(B) } 0.4 \qquad \textbf{(C) } 0.5 \qquad \textbf{(D) } 0.6 \qquad \textbf{(E) } 0.7</math>
+
<math>\textbf{(A) } \frac{24}{5} \qquad \textbf{(B) } \frac{42}{5} \qquad \textbf{(C) } \frac{81}{5} \qquad \textbf{(D) } 30 \qquad \textbf{(E) } 48</math>
  
[[2020 AMC 10A Problems/Problem 16|Solution]]
+
[[2022 AMC 10A Problems/Problem 16|Solution]]
  
 
==Problem 17==
 
==Problem 17==
  
Define<cmath>P(x) =(x-1^2)(x-2^2)\cdots(x-100^2).</cmath>How many integers <math>n</math> are there such that <math>P(n)\leq 0</math>?
+
How many three-digit positive integers <math>\underline{a} \ \underline{b} \ \underline{c}</math> are there whose nonzero digits <math>a,b,</math> and <math>c</math> satisfy
 +
<cmath>0.\overline{\underline{a}~\underline{b}~\underline{c}} = \frac{1}{3} (0.\overline{a} + 0.\overline{b} + 0.\overline{c})?</cmath>
 +
(The bar indicates repetition, thus <math>0.\overline{\underline{a}~\underline{b}~\underline{c}}</math> is the infinite repeating decimal <math>0.\underline{a}~\underline{b}~\underline{c}~\underline{a}~\underline{b}~\underline{c}~\cdots</math>)
  
<math>\textbf{(A) } 4900 \qquad \textbf{(B) } 4950\qquad \textbf{(C) } 5000\qquad \textbf{(D) } 5050 \qquad \textbf{(E) } 5100</math>
+
<math>\textbf{(A) } 9 \qquad \textbf{(B) } 10 \qquad \textbf{(C) } 11 \qquad \textbf{(D) } 13 \qquad \textbf{(E) } 14</math>
  
[[2020 AMC 10A Problems/Problem 17|Solution]]
+
[[2022 AMC 10A Problems/Problem 17|Solution]]
  
 
==Problem 18==
 
==Problem 18==
 +
Let <math>T_k</math> be the transformation of the coordinate plane that first rotates the plane <math>k</math> degrees counterclockwise around the origin and then reflects the plane across the <math>y</math>-axis. What is the least positive integer <math>n</math> such that performing the sequence of transformations <math>T_1, T_2, T_3,...,T_n</math> returns the point <math>(1, 0)</math> back to itself?
  
Let <math>(a,b,c,d)</math> be an ordered quadruple of not necessarily distinct integers, each one of them in the set <math>{0,1,2,3}.</math> For how many such quadruples is it true that <math>a\cdot d-b\cdot c</math> is odd? (For example, <math>(0,3,1,1)</math> is one such quadruple, because <math>0\cdot 1-3\cdot 1 = -3</math> is odd.)
+
<math>\textbf{(A) } 359 \qquad \textbf{(B) } 360 \qquad \textbf{(C) } 719 \qquad \textbf{(D) } 720 \qquad \textbf{(E) } 721 </math>
  
<math>\textbf{(A) } 48 \qquad \textbf{(B) } 64 \qquad \textbf{(C) } 96 \qquad \textbf{(D) } 128 \qquad \textbf{(E) } 192</math>
+
[[2022 AMC 10A Problems/Problem 18|Solution]]
  
[[2020 AMC 10A Problems/Problem 18|Solution]]
+
==Problem 19==
  
==Problem 19==
+
Define <math>L_n</math> as the least common multiple of all the integers from <math>1</math> to <math>n</math> inclusive. There is a unique integer <math>h</math> such that
 +
<cmath>\frac{1}{1}+\frac{1}{2}+\frac{1}{3}+\cdots+\frac{1}{17}=\frac{h}{L_{17}}</cmath>
 +
What is the remainder when <math>h</math> is divided by <math>17</math>?
  
As shown in the figure below, a regular dodecahedron (the polyhedron consisting of <math>12</math> congruent regular pentagonal faces) floats in empty space with two horizontal faces. Note that there is a ring of five slanted faces adjacent to the top face, and a ring of five slanted faces adjacent to the bottom face. How many ways are there to move from the top face to the bottom face via a sequence of adjacent faces so that each face is visited at most once and moves are not permitted from the bottom ring to the top ring?
+
<math>\textbf{(A) } 1 \qquad \textbf{(B) } 3 \qquad \textbf{(C) } 5 \qquad \textbf{(D) } 7 \qquad \textbf{(E) } 9</math>
<asy>
 
import graph;
 
unitsize(5cm);
 
pair A = (0.082, 0.378);
 
pair B = (0.091, 0.649);
 
pair C = (0.249, 0.899);
 
pair D = (0.479, 0.939);
 
pair E = (0.758, 0.893);
 
pair F = (0.862, 0.658);
 
pair G = (0.924, 0.403);
 
pair H = (0.747, 0.194);
 
pair I = (0.526, 0.075);
 
pair J = (0.251, 0.170);
 
pair K = (0.568, 0.234);
 
pair L = (0.262, 0.449);
 
pair M = (0.373, 0.813);
 
pair N = (0.731, 0.813);
 
pair O = (0.851, 0.461);
 
path[] f;
 
f[0] = A--B--C--M--L--cycle;
 
f[1] = C--D--E--N--M--cycle;
 
f[2] = E--F--G--O--N--cycle;
 
f[3] = G--H--I--K--O--cycle;
 
f[4] = I--J--A--L--K--cycle;
 
f[5] = K--L--M--N--O--cycle;
 
draw(f[0]);
 
axialshade(f[1], white, M, gray(0.5), (C+2*D)/3);
 
draw(f[1]);
 
filldraw(f[2], gray);
 
filldraw(f[3], gray);
 
axialshade(f[4], white, L, gray(0.7), J);
 
draw(f[4]);
 
draw(f[5]);
 
</asy>
 
<math>\textbf{(A) } 125 \qquad \textbf{(B) } 250 \qquad \textbf{(C) } 405 \qquad \textbf{(D) } 640 \qquad \textbf{(E) } 810</math>
 
  
[[2020 AMC 10A Problems/Problem 19|Solution]]
+
[[2022 AMC 10A Problems/Problem 19|Solution]]
  
 
==Problem 20==
 
==Problem 20==
  
Quadrilateral <math>ABCD</math> satisfies <math>\angle ABC = \angle ACD = 90^{\circ}, AC=20,</math> and <math>CD=30.</math> Diagonals <math>\overline{AC}</math> and <math>\overline{BD}</math> intersect at point <math>E,</math> and <math>AE=5.</math> What is the area of quadrilateral <math>ABCD?</math>
 
  
<math>\textbf{(A) } 330 \qquad \textbf{(B) } 340 \qquad \textbf{(C) } 350 \qquad \textbf{(D) } 360 \qquad \textbf{(E) } 370</math>
+
A four-term sequence is formed by adding each term of a four-term arithmetic sequence of positive integers to the corresponding term of a four-term geometric sequence of positive integers. The first three terms of the resulting four-term sequence are <math>57</math>, <math>60</math>, and <math>91</math>. What is the fourth term of this sequence?
 +
 
 +
<math>\textbf{(A) } 190 \qquad \textbf{(B) } 194 \qquad \textbf{(C) } 198 \qquad \textbf{(D) } 202 \qquad \textbf{(E) } 206</math>
  
[[2020 AMC 10A Problems/Problem 20|Solution]]
+
[[2022 AMC 10A Problems/Problem 20|Solution]]
  
 
==Problem 21==
 
==Problem 21==
  
There exists a unique strictly increasing sequence of nonnegative integers <math>a_1 < a_2 < … < a_k</math> such that<cmath>\frac{2^{289}+1}{2^{17}+1} = 2^{a_1} + 2^{a_2} + + 2^{a_k}.</cmath>What is <math>k?</math>
+
A bowl is formed by attaching four regular hexagons of side <math>1</math> to a square of side <math>1</math>. The edges of the adjacent hexagons coincide, as shown in the figure. What is the area of the octagon obtained by joining the top eight vertices of the four hexagons, situated on the rim of the bowl?
 +
<asy>
 +
import three;
 +
size(225);
 +
currentprojection=
 +
  orthographic(camera=(-5.52541796301147,-2.61548797564715,1.6545450372312),
 +
              up=(0.00247902062334861,0.000877141782387748,0.00966536329192992),
 +
              target=(0,0,0),
 +
              zoom=0.570588560870951);
 +
currentpen = black+1.5bp;
 +
triple A = O;
 +
triple M = (X+Y)/2;
 +
triple B = (-1/2,-1/2,1/sqrt(2));
 +
triple C = (-1,0,sqrt(2));
 +
triple D = (0,-1,sqrt(2));
 +
transform3 rho = rotate(90,M,M+Z);
 +
 
 +
//arrays of vertices for the lower level (the square), the middle level,
 +
//and the interleaves vertices of the upper level (the octagon)
 +
triple[] lVs = {A};
 +
triple[] mVs = {B};
 +
triple[] uVsl = {C};
 +
triple[] uVsr = {D};
 +
 
 +
for(int i = 0; i < 3; ++i){
 +
  lVs.push(rho*lVs[i]);
 +
  mVs.push(rho*mVs[i]);
 +
  uVsl.push(rho*uVsl[i]);
 +
  uVsr.push(rho*uVsr[i]);
 +
}
  
<math>\textbf{(A) } 117 \qquad \textbf{(B) } 136 \qquad \textbf{(C) } 137 \qquad \textbf{(D) } 273 \qquad \textbf{(E) } 306</math>
+
lVs.cyclic = true;
 +
uVsl.cyclic = true;
  
[[2020 AMC 10A Problems/Problem 21|Solution]]
+
for(int i : new int[] {0,1,2,3}){
 +
  draw(uVsl[i]--uVsr[i]);
 +
  draw(uVsr[i]--uVsl[i+1]);
 +
}
 +
draw(lVs[0]--lVs[1]^^lVs[0]--lVs[3]);
 +
for(int i : new int[] {0,1,3}){
 +
  draw(lVs[0]--lVs[i]);
 +
  draw(lVs[i]--mVs[i]);
 +
  draw(mVs[i]--uVsl[i]);
 +
}
 +
for(int i : new int[] {0,3}){
 +
  draw(mVs[i]--uVsr[i]);
 +
}
 +
 
 +
for(int i : new int[] {1,3}) draw(lVs[2]--lVs[i],dashed);
 +
draw(lVs[2]--mVs[2],dashed);
 +
draw(mVs[2]--uVsl[2]^^mVs[2]--uVsr[2],dashed);
 +
draw(mVs[1]--uVsr[1],dashed);
 +
 
 +
//Comment two lines below to remove red edges
 +
//draw(lVs[1]--lVs[3],red+2bp);
 +
//draw(uVsl[0]--uVsr[0],red+2bp);
 +
</asy>
 +
<math>\textbf{(A) } 6 \qquad \textbf{(B) } 7 \qquad \textbf{(C) } 5+2\sqrt{2} \qquad \textbf{(D) } 8 \qquad \textbf{(E) } 9</math>
 +
 
 +
[[2022 AMC 10A Problems/Problem 21|Solution]]
  
 
==Problem 22==
 
==Problem 22==
  
For how many positive integers <math>n \le 1000</math> is<cmath>\left\lfloor \dfrac{998}{n} \right\rfloor+\left\lfloor \dfrac{999}{n} \right\rfloor+\left\lfloor \dfrac{1000}{n}\right \rfloor</cmath>not divisible by <math>3</math>? (Recall that <math>\lfloor x \rfloor</math> is the greatest integer less than or equal to <math>x</math>.)
+
Suppose that <math>13</math> cards numbered <math>1, 2, 3, \ldots, 13</math> are arranged in a row. The task is to pick them up in numerically increasing order, working repeatedly from left to right. In the example below, cards <math>1, 2, 3</math> are picked up on the first pass, <math>4</math> and <math>5</math> on the second pass, <math>6</math> on the third pass, <math>7, 8, 9, 10</math> on the fourth pass, and <math>11, 12, 13</math> on the fifth pass. For how many of the <math>13!</math> possible orderings of the cards will the <math>13</math> cards be picked up in exactly two passes?
  
<math>\textbf{(A) } 22 \qquad\textbf{(B) } 23 \qquad\textbf{(C) } 24 \qquad\textbf{(D) } 25 \qquad\textbf{(E) } 26</math>
+
<asy>
 +
size(11cm);
 +
draw((0,0)--(2,0)--(2,3)--(0,3)--cycle);
 +
label("7", (1,1.5));
 +
draw((3,0)--(5,0)--(5,3)--(3,3)--cycle);
 +
label("11", (4,1.5));
 +
draw((6,0)--(8,0)--(8,3)--(6,3)--cycle);
 +
label("8", (7,1.5));
 +
draw((9,0)--(11,0)--(11,3)--(9,3)--cycle);
 +
label("6", (10,1.5));
 +
draw((12,0)--(14,0)--(14,3)--(12,3)--cycle);
 +
label("4", (13,1.5));
 +
draw((15,0)--(17,0)--(17,3)--(15,3)--cycle);
 +
label("5", (16,1.5));
 +
draw((18,0)--(20,0)--(20,3)--(18,3)--cycle);
 +
label("9", (19,1.5));
 +
draw((21,0)--(23,0)--(23,3)--(21,3)--cycle);
 +
label("12", (22,1.5));
 +
draw((24,0)--(26,0)--(26,3)--(24,3)--cycle);
 +
label("1", (25,1.5));
 +
draw((27,0)--(29,0)--(29,3)--(27,3)--cycle);
 +
label("13", (28,1.5));
 +
draw((30,0)--(32,0)--(32,3)--(30,3)--cycle);
 +
label("10", (31,1.5));
 +
draw((33,0)--(35,0)--(35,3)--(33,3)--cycle);
 +
label("2", (34,1.5));
 +
draw((36,0)--(38,0)--(38,3)--(36,3)--cycle);
 +
label("3", (37,1.5));
 +
</asy>
 +
<math>\textbf{(A) } 4082 \qquad \textbf{(B) } 4095 \qquad \textbf{(C) } 4096 \qquad \textbf{(D) } 8178 \qquad \textbf{(E) } 8191</math>
  
[[2020 AMC 10A Problems/Problem 22|Solution]]
+
[[2022 AMC 10A Problems/Problem 22|Solution]]
  
 
==Problem 23==
 
==Problem 23==
  
Let <math>T</math> be the triangle in the coordinate plane with vertices <math>(0,0), (4,0),</math> and <math>(0,3).</math> Consider the following five isometries (rigid transformations) of the plane: rotations of <math>90^{\circ}, 180^{\circ},</math> and <math>270^{\circ}</math> counterclockwise around the origin, reflection across the <math>x</math>-axis, and reflection across the <math>y</math>-axis. How many of the <math>125</math> sequences of three of these transformations (not necessarily distinct) will return <math>T</math> to its original position? (For example, a <math>180^{\circ}</math> rotation, followed by a reflection across the <math>x</math>-axis, followed by a reflection across the <math>y</math>-axis will return <math>T</math> to its original position, but a <math>90^{\circ}</math> rotation, followed by a reflection across the <math>x</math>-axis, followed by another reflection across the <math>x</math>-axis will not return <math>T</math> to its original position.)
+
Isosceles trapezoid <math>ABCD</math> has parallel sides <math>\overline{AD}</math> and <math>\overline{BC},</math> with <math>BC < AD</math> and <math>AB = CD.</math> There is a point <math>P</math> in the plane such that <math>PA=1, PB=2, PC=3,</math> and <math>PD=4.</math> What is <math>\tfrac{BC}{AD}?</math>
  
<math>\textbf{(A) } 12 \qquad \textbf{(B) } 15 \qquad \textbf{(C) } 17 \qquad \textbf{(D) } 20 \qquad \textbf{(E) } 25</math>
+
<math>\textbf{(A) }\frac{1}{4}\qquad\textbf{(B) }\frac{1}{3}\qquad\textbf{(C) }\frac{1}{2}\qquad\textbf{(D) }\frac{2}{3}\qquad\textbf{(E) }\frac{3}{4}</math>
  
[[2020 AMC 10A Problems/Problem 23|Solution]]
+
[[2022 AMC 10A Problems/Problem 23|Solution]]
  
 
==Problem 24==
 
==Problem 24==
  
Let <math>n</math> be the least positive integer greater than <math>1000</math> for which<cmath>\gcd(63, n+120) =21\quad \text{and} \quad \gcd(n+63, 120)=60.</cmath>What is the sum of the digits of <math>n</math>?
+
How many strings of length <math>5</math> formed from the digits <math>0</math>, <math>1</math>, <math>2</math>, <math>3</math>, <math>4</math> are there such that for each <math>j \in \{1,2,3,4\}</math>, at least <math>j</math> of the digits are less than <math>j</math>? (For example, <math>02214</math> satisfies this condition
 +
because it contains at least <math>1</math> digit less than <math>1</math>, at least <math>2</math> digits less than <math>2</math>, at least <math>3</math> digits less
 +
than <math>3</math>, and at least <math>4</math> digits less than <math>4</math>. The string <math>23404</math> does not satisfy the condition because it
 +
does not contain at least <math>2</math> digits less than <math>2</math>.)
  
<math>\textbf{(A) } 12 \qquad\textbf{(B) } 15 \qquad\textbf{(C) } 18 \qquad\textbf{(D) } 21\qquad\textbf{(E) } 24</math>
+
<math>\textbf{(A) }500\qquad\textbf{(B) }625\qquad\textbf{(C) }1089\qquad\textbf{(D) }1199\qquad\textbf{(E) }1296</math>
  
[[2020 AMC 10A Problems/Problem 24|Solution]]
+
[[2022 AMC 10A Problems/Problem 24|Solution]]
  
 
==Problem 25==
 
==Problem 25==
  
Jason rolls three fair standard six-sided dice. Then he looks at the rolls and chooses a subset of the dice (possibly empty, possibly all three dice) to reroll. After rerolling, he wins if and only if the sum of the numbers face up on the three dice is exactly <math>7.</math> Jason always plays to optimize his chances of winning. What is the probability that he chooses to reroll exactly two of the dice?
+
Let <math>R</math>, <math>S</math>, and <math>T</math> be squares that have vertices at lattice points (i.e., points whose coordinates are both integers) in the coordinate plane, together with their interiors. The bottom edge of each square is on the <math>x</math>-axis. The left edge of <math>R</math> and the right edge of <math>S</math> are on the <math>y</math>-axis, and <math>R</math> contains <math>\frac{9}{4}</math> as many lattice points as does <math>S</math>. The top two vertices of <math>T</math> are in <math>R \cup S</math>, and <math>T</math> contains <math>\frac{1}{4}</math> of the lattice points contained in <math>R \cup S.</math> See the figure (not drawn to scale).
 +
<asy>
 +
size(8cm);
 +
label(scale(.8)*"$y$", (0,60), N);
 +
label(scale(.8)*"$x$", (60,0), E);
 +
filldraw((0,0)--(55,0)--(55,55)--(0,55)--cycle, yellow+orange+white+white);
 +
label(scale(1.3)*"$R$", (55/2,55/2));
 +
filldraw((0,0)--(0,28)--(-28,28)--(-28,0)--cycle, green+white+white);
 +
label(scale(1.3)*"$S$",(-14,14));
 +
filldraw((-10,0)--(15,0)--(15,25)--(-10,25)--cycle, red+white+white);
 +
label(scale(1.3)*"$T$",(3.5,25/2));
 +
draw((0,-10)--(0,60),EndArrow());
 +
draw((-34,0)--(60,0),EndArrow());
 +
</asy>
 +
The fraction of lattice points in <math>S</math> that are in <math>S \cap T</math> is <math>27</math> times the fraction of lattice points in <math>R</math> that are in <math>R \cap T</math>. What is the minimum possible value of the edge length of <math>R</math> plus the edge length of <math>S</math> plus the edge length of <math>T</math>?
  
<math>\textbf{(A) } \frac{7}{36} \qquad\textbf{(B) } \frac{5}{24} \qquad\textbf{(C) } \frac{2}{9} \qquad\textbf{(D) } \frac{17}{72} \qquad\textbf{(E) } \frac{1}{4}</math>
+
<math>\textbf{(A) }336\qquad\textbf{(B) }337\qquad\textbf{(C) }338\qquad\textbf{(D) }339\qquad\textbf{(E) }340</math>
  
[[2020 AMC 10A Problems/Problem 25|Solution]]
+
[[2022 AMC 10A Problems/Problem 25|Solution]]
  
 
==See also==
 
==See also==
{{AMC10 box|year=2020|ab=A|before=[[2019 AMC 10B Problems]]|after=[[2020 AMC 10B Problems]]}}
+
{{AMC10 box|year=2022|ab=A|before=[[2021 Fall AMC 10B Problems]]|after=[[2022 AMC 10B Problems]]}}
 
{{MAA Notice}}
 
{{MAA Notice}}

Latest revision as of 02:08, 9 March 2024

2022 AMC 10A (Answer Key)
Printable versions: WikiAoPS ResourcesPDF

Instructions

  1. This is a 25-question, multiple choice test. Each question is followed by answers marked A, B, C, D and E. Only one of these is correct.
  2. You will receive 6 points for each correct answer, 2.5 points for each problem left unanswered if the year is before 2006, 1.5 points for each problem left unanswered if the year is after 2006, and 0 points for each incorrect answer.
  3. No aids are permitted other than scratch paper, graph paper, ruler, compass, protractor and erasers (and calculators that are accepted for use on the SAT if before 2006. No problems on the test will require the use of a calculator).
  4. Figures are not necessarily drawn to scale.
  5. You will have 75 minutes working time to complete the test.
1 2 3 4 5 6 7 8 9 10 11 12 13 14 15 16 17 18 19 20 21 22 23 24 25

Problem 1

What is the value of \[3+\frac{1}{3+\frac{1}{3+\frac13}}?\] $\textbf{(A)}\ \frac{31}{10}\qquad\textbf{(B)}\ \frac{49}{15}\qquad\textbf{(C)}\ \frac{33}{10}\qquad\textbf{(D)}\ \frac{109}{33}\qquad\textbf{(E)}\ \frac{15}{4}$

Solution

Problem 2

Mike cycled $15$ laps in $57$ minutes. Assume he cycled at a constant speed throughout. Approximately how many laps did he complete in the first $27$ minutes?

$\textbf{(A) } 5 \qquad\textbf{(B) } 7 \qquad\textbf{(C) } 9 \qquad\textbf{(D) } 11 \qquad\textbf{(E) } 13$

Solution

Problem 3

The sum of three numbers is $96.$ The first number is $6$ times the third number, and the third number is $40$ less than the second number. What is the absolute value of the difference between the first and second numbers?

$\textbf{(A) } 1 \qquad \textbf{(B) } 2 \qquad \textbf{(C) } 3 \qquad \textbf{(D) } 4 \qquad \textbf{(E) } 5$

Solution

Problem 4

In some countries, automobile fuel efficiency is measured in liters per $100$ kilometers while other countries use miles per gallon. Suppose that 1 kilometer equals $m$ miles, and $1$ gallon equals $l$ liters. Which of the following gives the fuel efficiency in liters per $100$ kilometers for a car that gets $x$ miles per gallon?

$\textbf{(A) } \frac{x}{100lm} \qquad \textbf{(B) } \frac{xlm}{100} \qquad \textbf{(C) } \frac{lm}{100x} \qquad \textbf{(D) } \frac{100}{xlm} \qquad \textbf{(E) } \frac{100lm}{x}$

Solution

Problem 5

Square $ABCD$ has side length $1$. Points $P$, $Q$, $R$, and $S$ each lie on a side of $ABCD$ such that $APQCRS$ is an equilateral convex hexagon with side length $s$. What is $s$?

$\textbf{(A) } \frac{\sqrt{2}}{3} \qquad \textbf{(B) } \frac{1}{2} \qquad \textbf{(C) } 2 - \sqrt{2} \qquad \textbf{(D) } 1 - \frac{\sqrt{2}}{4} \qquad \textbf{(E) } \frac{2}{3}$

Solution

Problem 6

Which expression is equal to \[\left|a-2-\sqrt{(a-1)^2}\right|\] for $a<0?$

$\textbf{(A) } 3-2a \qquad \textbf{(B) } 1-a \qquad \textbf{(C) } 1 \qquad \textbf{(D) } a+1 \qquad \textbf{(E) } 3$

Solution

Problem 7

The least common multiple of a positive integer $n$ and $18$ is $180$, and the greatest common divisor of $n$ and $45$ is $15$. What is the sum of the digits of $n$?

$\textbf{(A) } 3 \qquad \textbf{(B) } 6 \qquad \textbf{(C) } 8 \qquad \textbf{(D) } 9 \qquad \textbf{(E) } 12$

Solution

Problem 8

A data set consists of $6$ (not distinct) positive integers: $1$, $7$, $5$, $2$, $5$, and $X$. The average (arithmetic mean) of the $6$ numbers equals a value in the data set. What is the sum of all possible values of $X$?

$\textbf{(A) } 10 \qquad \textbf{(B) } 26 \qquad \textbf{(C) } 32 \qquad \textbf{(D) } 36 \qquad \textbf{(E) } 40$

Solution

Problem 9

A rectangle is partitioned into $5$ regions as shown. Each region is to be painted a solid color - red, orange, yellow, blue, or green - so that regions that touch are painted different colors, and colors can be used more than once. How many different colorings are possible?

[asy] size(5.5cm); draw((0,0)--(0,2)--(2,2)--(2,0)--cycle); draw((2,0)--(8,0)--(8,2)--(2,2)--cycle); draw((8,0)--(12,0)--(12,2)--(8,2)--cycle); draw((0,2)--(6,2)--(6,4)--(0,4)--cycle); draw((6,2)--(12,2)--(12,4)--(6,4)--cycle); [/asy]

$\textbf{(A) }120\qquad\textbf{(B) }270\qquad\textbf{(C) }360\qquad\textbf{(D) }540\qquad\textbf{(E) }720$

Solution

Problem 10

Daniel finds a rectangular index card and measures its diagonal to be $8$ centimeters. Daniel then cuts out equal squares of side $1$ cm at two opposite corners of the index card and measures the distance between the two closest vertices of these squares to be $4\sqrt{2}$ centimeters, as shown below. What is the area of the original index card? [asy] // Diagram by MRENTHUSIASM, edited by Djmathman size(200); defaultpen(linewidth(0.6)); draw((489.5,-213) -- (225.5,-213) -- (225.5,-185) -- (199.5,-185) -- (198.5,-62) -- (457.5,-62) -- (457.5,-93) -- (489.5,-93) -- cycle); draw((206.29,-70.89) -- (480.21,-207.11), linetype ("6 6"),Arrows(size=4,arrowhead=HookHead)); draw((237.85,-182.24) -- (448.65,-95.76),linetype ("6 6"),Arrows(size=4,arrowhead=HookHead)); label("$1$",(450,-80)); label("$1$",(475,-106)); label("$8$",(300,-103)); label("$4\sqrt 2$",(300,-173)); [/asy] $\textbf{(A) } 14 \qquad \textbf{(B) } 10\sqrt{2} \qquad \textbf{(C) } 16 \qquad \textbf{(D) } 12\sqrt{2} \qquad \textbf{(E) } 18$

Solution

Problem 11

Ted mistakenly wrote $2^m\cdot\sqrt{\frac{1}{4096}}$ as $2\cdot\sqrt[m]{\frac{1}{4096}}.$ What is the sum of all real numbers $m$ for which these two expressions have the same value?

$\textbf{(A) } 5 \qquad \textbf{(B) } 6 \qquad \textbf{(C) } 7 \qquad \textbf{(D) } 8 \qquad \textbf{(E) } 9$

Solution

Problem 12

On Halloween $31$ children walked into the principal's office asking for candy. They can be classified into three types: Some always lie; some always tell the truth; and some alternately lie and tell the truth. The alternaters arbitrarily choose their first response, either a lie or the truth, but each subsequent statement has the opposite truth value from its predecessor. The principal asked everyone the same three questions in this order.

"Are you a truth-teller?" The principal gave a piece of candy to each of the $22$ children who answered yes.

"Are you an alternater?" The principal gave a piece of candy to each of the $15$ children who answered yes.

"Are you a liar?" The principal gave a piece of candy to each of the $9$ children who answered yes.

How many pieces of candy in all did the principal give to the children who always tell the truth?

$\textbf{(A) } 7 \qquad \textbf{(B) } 12 \qquad \textbf{(C) } 21 \qquad \textbf{(D) } 27 \qquad \textbf{(E) } 31$

Solution

Problem 13

Let $\triangle ABC$ be a scalene triangle. Point $P$ lies on $\overline{BC}$ so that $\overline{AP}$ bisects $\angle BAC.$ The line through $B$ perpendicular to $\overline{AP}$ intersects the line through $A$ parallel to $\overline{BC}$ at point $D.$ Suppose $BP=2$ and $PC=3.$ What is $AD?$

$\textbf{(A) } 8 \qquad \textbf{(B) } 9 \qquad \textbf{(C) } 10 \qquad \textbf{(D) } 11 \qquad \textbf{(E) } 12$

Solution

Problem 14

How many ways are there to split the integers $1$ through $14$ into $7$ pairs such that in each pair, the greater number is at least $2$ times the lesser number?

$\textbf{(A) } 108 \qquad \textbf{(B) } 120 \qquad \textbf{(C) } 126 \qquad \textbf{(D) } 132 \qquad \textbf{(E) } 144$

Solution

Problem 15

Quadrilateral $ABCD$ with side lengths $AB=7, BC=24, CD=20, DA=15$ is inscribed in a circle. The area interior to the circle but exterior to the quadrilateral can be written in the form $\frac{a\pi-b}{c},$ where $a,b,$ and $c$ are positive integers such that $a$ and $c$ have no common prime factor. What is $a+b+c?$

$\textbf{(A) } 260 \qquad \textbf{(B) } 855 \qquad \textbf{(C) } 1235 \qquad \textbf{(D) } 1565 \qquad \textbf{(E) } 1997$

Solution

Problem 16

The roots of the polynomial $10x^3 - 39x^2 + 29x - 6$ are the height, length, and width of a rectangular box (right rectangular prism). A new rectangular box is formed by lengthening each edge of the original box by $2$ units. What is the volume of the new box?

$\textbf{(A) } \frac{24}{5} \qquad \textbf{(B) } \frac{42}{5} \qquad \textbf{(C) } \frac{81}{5} \qquad \textbf{(D) } 30 \qquad \textbf{(E) } 48$

Solution

Problem 17

How many three-digit positive integers $\underline{a} \ \underline{b} \ \underline{c}$ are there whose nonzero digits $a,b,$ and $c$ satisfy \[0.\overline{\underline{a}~\underline{b}~\underline{c}} = \frac{1}{3} (0.\overline{a} + 0.\overline{b} + 0.\overline{c})?\] (The bar indicates repetition, thus $0.\overline{\underline{a}~\underline{b}~\underline{c}}$ is the infinite repeating decimal $0.\underline{a}~\underline{b}~\underline{c}~\underline{a}~\underline{b}~\underline{c}~\cdots$)

$\textbf{(A) } 9 \qquad \textbf{(B) } 10 \qquad \textbf{(C) } 11 \qquad \textbf{(D) } 13 \qquad \textbf{(E) } 14$

Solution

Problem 18

Let $T_k$ be the transformation of the coordinate plane that first rotates the plane $k$ degrees counterclockwise around the origin and then reflects the plane across the $y$-axis. What is the least positive integer $n$ such that performing the sequence of transformations $T_1, T_2, T_3,...,T_n$ returns the point $(1, 0)$ back to itself?

$\textbf{(A) } 359 \qquad \textbf{(B) } 360 \qquad \textbf{(C) } 719 \qquad \textbf{(D) } 720 \qquad \textbf{(E) } 721$

Solution

Problem 19

Define $L_n$ as the least common multiple of all the integers from $1$ to $n$ inclusive. There is a unique integer $h$ such that \[\frac{1}{1}+\frac{1}{2}+\frac{1}{3}+\cdots+\frac{1}{17}=\frac{h}{L_{17}}\] What is the remainder when $h$ is divided by $17$?

$\textbf{(A) } 1 \qquad \textbf{(B) } 3 \qquad \textbf{(C) } 5 \qquad \textbf{(D) } 7 \qquad \textbf{(E) } 9$

Solution

Problem 20

A four-term sequence is formed by adding each term of a four-term arithmetic sequence of positive integers to the corresponding term of a four-term geometric sequence of positive integers. The first three terms of the resulting four-term sequence are $57$, $60$, and $91$. What is the fourth term of this sequence?

$\textbf{(A) } 190 \qquad \textbf{(B) } 194 \qquad \textbf{(C) } 198 \qquad \textbf{(D) } 202 \qquad \textbf{(E) } 206$

Solution

Problem 21

A bowl is formed by attaching four regular hexagons of side $1$ to a square of side $1$. The edges of the adjacent hexagons coincide, as shown in the figure. What is the area of the octagon obtained by joining the top eight vertices of the four hexagons, situated on the rim of the bowl? [asy] import three; size(225); currentprojection=   orthographic(camera=(-5.52541796301147,-2.61548797564715,1.6545450372312),                up=(0.00247902062334861,0.000877141782387748,0.00966536329192992),                target=(0,0,0),                zoom=0.570588560870951); currentpen = black+1.5bp; triple A = O; triple M = (X+Y)/2; triple B = (-1/2,-1/2,1/sqrt(2)); triple C = (-1,0,sqrt(2)); triple D = (0,-1,sqrt(2)); transform3 rho = rotate(90,M,M+Z);  //arrays of vertices for the lower level (the square), the middle level, //and the interleaves vertices of the upper level (the octagon) triple[] lVs = {A}; triple[] mVs = {B}; triple[] uVsl = {C}; triple[] uVsr = {D};  for(int i = 0; i < 3; ++i){   lVs.push(rho*lVs[i]);   mVs.push(rho*mVs[i]);   uVsl.push(rho*uVsl[i]);   uVsr.push(rho*uVsr[i]); }  lVs.cyclic = true; uVsl.cyclic = true;  for(int i : new int[] {0,1,2,3}){   draw(uVsl[i]--uVsr[i]);   draw(uVsr[i]--uVsl[i+1]); } draw(lVs[0]--lVs[1]^^lVs[0]--lVs[3]); for(int i : new int[] {0,1,3}){   draw(lVs[0]--lVs[i]);   draw(lVs[i]--mVs[i]);   draw(mVs[i]--uVsl[i]); } for(int i : new int[] {0,3}){   draw(mVs[i]--uVsr[i]); }  for(int i : new int[] {1,3}) draw(lVs[2]--lVs[i],dashed); draw(lVs[2]--mVs[2],dashed); draw(mVs[2]--uVsl[2]^^mVs[2]--uVsr[2],dashed); draw(mVs[1]--uVsr[1],dashed);  //Comment two lines below to remove red edges //draw(lVs[1]--lVs[3],red+2bp); //draw(uVsl[0]--uVsr[0],red+2bp); [/asy] $\textbf{(A) } 6 \qquad \textbf{(B) } 7 \qquad \textbf{(C) } 5+2\sqrt{2} \qquad \textbf{(D) } 8 \qquad \textbf{(E) } 9$

Solution

Problem 22

Suppose that $13$ cards numbered $1, 2, 3, \ldots, 13$ are arranged in a row. The task is to pick them up in numerically increasing order, working repeatedly from left to right. In the example below, cards $1, 2, 3$ are picked up on the first pass, $4$ and $5$ on the second pass, $6$ on the third pass, $7, 8, 9, 10$ on the fourth pass, and $11, 12, 13$ on the fifth pass. For how many of the $13!$ possible orderings of the cards will the $13$ cards be picked up in exactly two passes?

[asy] size(11cm); draw((0,0)--(2,0)--(2,3)--(0,3)--cycle); label("7", (1,1.5)); draw((3,0)--(5,0)--(5,3)--(3,3)--cycle); label("11", (4,1.5)); draw((6,0)--(8,0)--(8,3)--(6,3)--cycle); label("8", (7,1.5)); draw((9,0)--(11,0)--(11,3)--(9,3)--cycle); label("6", (10,1.5)); draw((12,0)--(14,0)--(14,3)--(12,3)--cycle); label("4", (13,1.5)); draw((15,0)--(17,0)--(17,3)--(15,3)--cycle); label("5", (16,1.5)); draw((18,0)--(20,0)--(20,3)--(18,3)--cycle); label("9", (19,1.5)); draw((21,0)--(23,0)--(23,3)--(21,3)--cycle); label("12", (22,1.5)); draw((24,0)--(26,0)--(26,3)--(24,3)--cycle); label("1", (25,1.5)); draw((27,0)--(29,0)--(29,3)--(27,3)--cycle); label("13", (28,1.5)); draw((30,0)--(32,0)--(32,3)--(30,3)--cycle); label("10", (31,1.5)); draw((33,0)--(35,0)--(35,3)--(33,3)--cycle); label("2", (34,1.5)); draw((36,0)--(38,0)--(38,3)--(36,3)--cycle); label("3", (37,1.5)); [/asy] $\textbf{(A) } 4082 \qquad \textbf{(B) } 4095 \qquad \textbf{(C) } 4096 \qquad \textbf{(D) } 8178 \qquad \textbf{(E) } 8191$

Solution

Problem 23

Isosceles trapezoid $ABCD$ has parallel sides $\overline{AD}$ and $\overline{BC},$ with $BC < AD$ and $AB = CD.$ There is a point $P$ in the plane such that $PA=1, PB=2, PC=3,$ and $PD=4.$ What is $\tfrac{BC}{AD}?$

$\textbf{(A) }\frac{1}{4}\qquad\textbf{(B) }\frac{1}{3}\qquad\textbf{(C) }\frac{1}{2}\qquad\textbf{(D) }\frac{2}{3}\qquad\textbf{(E) }\frac{3}{4}$

Solution

Problem 24

How many strings of length $5$ formed from the digits $0$, $1$, $2$, $3$, $4$ are there such that for each $j \in \{1,2,3,4\}$, at least $j$ of the digits are less than $j$? (For example, $02214$ satisfies this condition because it contains at least $1$ digit less than $1$, at least $2$ digits less than $2$, at least $3$ digits less than $3$, and at least $4$ digits less than $4$. The string $23404$ does not satisfy the condition because it does not contain at least $2$ digits less than $2$.)

$\textbf{(A) }500\qquad\textbf{(B) }625\qquad\textbf{(C) }1089\qquad\textbf{(D) }1199\qquad\textbf{(E) }1296$

Solution

Problem 25

Let $R$, $S$, and $T$ be squares that have vertices at lattice points (i.e., points whose coordinates are both integers) in the coordinate plane, together with their interiors. The bottom edge of each square is on the $x$-axis. The left edge of $R$ and the right edge of $S$ are on the $y$-axis, and $R$ contains $\frac{9}{4}$ as many lattice points as does $S$. The top two vertices of $T$ are in $R \cup S$, and $T$ contains $\frac{1}{4}$ of the lattice points contained in $R \cup S.$ See the figure (not drawn to scale). [asy] size(8cm); label(scale(.8)*"$y$", (0,60), N); label(scale(.8)*"$x$", (60,0), E); filldraw((0,0)--(55,0)--(55,55)--(0,55)--cycle, yellow+orange+white+white); label(scale(1.3)*"$R$", (55/2,55/2)); filldraw((0,0)--(0,28)--(-28,28)--(-28,0)--cycle, green+white+white); label(scale(1.3)*"$S$",(-14,14)); filldraw((-10,0)--(15,0)--(15,25)--(-10,25)--cycle, red+white+white); label(scale(1.3)*"$T$",(3.5,25/2)); draw((0,-10)--(0,60),EndArrow()); draw((-34,0)--(60,0),EndArrow()); [/asy] The fraction of lattice points in $S$ that are in $S \cap T$ is $27$ times the fraction of lattice points in $R$ that are in $R \cap T$. What is the minimum possible value of the edge length of $R$ plus the edge length of $S$ plus the edge length of $T$?

$\textbf{(A) }336\qquad\textbf{(B) }337\qquad\textbf{(C) }338\qquad\textbf{(D) }339\qquad\textbf{(E) }340$

Solution

See also

2022 AMC 10A (ProblemsAnswer KeyResources)
Preceded by
2021 Fall AMC 10B Problems
Followed by
2022 AMC 10B Problems
1 2 3 4 5 6 7 8 9 10 11 12 13 14 15 16 17 18 19 20 21 22 23 24 25
All AMC 10 Problems and Solutions

The problems on this page are copyrighted by the Mathematical Association of America's American Mathematics Competitions. AMC logo.png